Help whats the answer

Help Whats The Answer

Answers

Answer 1

Answer:

Company B, since 40 > 25 and 40 > 35

Step-by-step explanation:

Company A:

With each window more, price increases by 25

Company B:

15, 55, 95, 135, 175

With each window, the price increases by 40

Company C:

y = 35x + 25

x = number of windows

If x increases by 1, the price increases by 35, therefore, this is the increase per window

Answer 2

Answer:

B

Step-by-step explanation:


Related Questions

Given the table of the following quadratic function, which statement about this function is NOT true?

A. The y-intercept of f(x) is 6.
B. The zeros of the function f(x) are -3 and 1.
C. The minimum of the function f(x) is 8.
D. The axis of symmetry of the function f(x) is x = -1.

Answers

C is the incorrect answer

Note: Enter your answer and show all the steps that you use to solve this problem in the space provided.

Solve the following equation.
U + 2 = 4

Answers

Answer:

U=2

Step-by-step explanation:

This is a simple one-step algebraic equation. In algebra to solve equations, you should use the properties of equality. The properties say that an equation remains true as long as the same thing happens on both sides. For this equation, the property of subtracting should be used; this property states that the same number can be subtracted from both sides. To find U, isolate the variable by subtracting 2 from both sides. This leaves U=2.

U +2= 4
-2 -2
U = 2
2 + 2= 4

Hi this is 7th grade geometry with screenshot

Answers

Answer:

80

Step-by-step explanation:

ㄥAEC+160=180

ㄥAEC=20

ㄥAEC+x+80=180

20+x+80=180

x=80

There is a more simple method

x+80=160

x=80

It’s 80 because it is and it’s right

(trigonometry question)
-I'll give branliest as a reward-

Answers

Answer:

AC=2.868

Step-by-step explanation:

using pythagoras theorem

take 35 degree as reference angle

hypotenuse =5 , opposite =AC , adjacent=BC

using sin rule

sin 35=opposite/hypotenuse

0.5735=AC/5

0.5735*5=AC

2.8675=AC

2.868=AC

Ac=2.868 sorry if I am wrong

PLS HELP ME
Fine the circumference

Answers

Bgggvhhgfb dgfvvvcfvv

Answer:

2. 153.93804

Step-by-step explanation:

7 squared times pi is 153.93804

Simplify. 3√3 /√30
step by step

Answers

The answer is 0.9486. See the attachment

Multiply the numerator and denominator by the conjugate.

A conjugate is the inverse of an operation e. g + is the conjugate of -.

therefore the answer is squareroot of 10 divided by 3.

Help please?!!!!!!!!:8/8/&

Answers

Answer:

The answer is A)  y =3x + 3

Step-by-step explanation:

You substitute the gradient and y intercept into the equation of y = mx +b

the answer is A.) y=3x+3

The ratio of the corresponding linear measures of two similar waffle cones is 2 to 3. The smaller cone has a volume of about 18 cubic inches.

Answers

Answer:

what is joint stock company

Is this what you are looking for?
42.7 in ^3

which angles are complementary to 3 ? Select all that apply

Answers

Answer:

4

Step-by-step explanation:

4 is the answer………………..

which angle is adjacent to 9 ?

Answers

Adjacent means next to, so 12

Answer:

its the last one 12

Step-by-step explanation:

Find the value of x please.

Answers

Answer:

83

Step-by-step explanation:

its a guess bc its a right triangle and right triangle is 90 degrees minus 7 is 83

Answer:

I think it is 93 degrees

Step-by-step explanation:

A triangle has a full angle of 180 degrees

47+47 = 94

180-94=86

x-7=86

x = 93

Graph the points (0, -2), (4, 0) and (2, 0). If you want to create a parallelogram, which of the following points should be used for the fourth point?
A. (2, -2)
B. (-1, 0)
C. (-4, 0)
D. (4, 4)

Answers

It’s going to be c because it hits (-4,0)
use c for the fourth point because it is (-4,0)

Example Given


Find a or b

a = 20. b = ? c = 25



Find b or a. c^2 - a^2 = b^2



25^2 - 20^2 = c^2

625 - 400 = 225

Square Root of 225 is 15


QUESTION

a = 3 b = 4 c = ?

Answers

the answer is 5

a^2+b^2=c^2
9+16=25
sq root of 25 is 5

Answer:

the answer is 5

Step-by-step explanation:

Please help screen shot bellow.

Answers

-1/4

This will remain its absolute value

Answer:

absolute value is the oppistie of the number so if it is positive is will turn negative so i think the answer is C

PLEASE HELP ASAP!
Consider the 720 permutations of the numbers 1, 2, 3, 4, 5, 6. In how many of these permutations do exactly 3 of the numbers end up in the positions where they started?

Answers

The set {1,2,3,4,5,6} has a total of 6! permutations

a. Of those 6! permutations, 5!=120 begin with 1. So first 120 numbers would contain 1 as the unit digit.

b. The next 120, including the 124th, would begin with 2.

c. Then of the 5! numbers beginning with 2, there are 4!=24 including the 124th number, which have the second digit =1

d. Of these 4! permutations beginning with 21, there are 3!=6 including the 124th permutation which have third digit 3

e. Among these 3! permutations beginning with 213, there are 2 numbers with the fourth digit =4 (121th & 122th), 2 with fourth digit 5 (numbers 123 & 124) and 2 with fourth digit 6 (numbers 125 and 126).

Lastly, of the 2! permutations beginning with 2135, there is one with 5th digit 4 (number 123) and one with 5 digit 6 (number 124).

∴ The 124th number is 213564

Similarly reversing the above procedure we can determine the position of 321546 to be 267th on the list.

The set {1,2,3,4,5,6} has a total of 6! permutations a. Of those 6! permutations, 5!=120 begin with 1. So first 120 numbers would contain 1 as the unit digit. B. The next 120, including the 124th, would begin with 2. c. Then of the 5! numbers beginning with 2, there are 4!=24 including the 124th number, which have the second digit =1 d. Of these 4! permutations beginning with 21, there are 3!=6 including the 124th permutation which have third digit 3 e. Among these 3! permutations beginning with 213, there are 2 numbers with the fourth digit =4 (121th & 122th), 2 with fourth digit 5 ​​(numbers 123 & 124) and 2 with fourth digit 6 (numbers 125 and 126). Lastly, of the 2! permutations beginning with 2135, there is one with 5th digit 4 (number 123) and

PLS HELP MY GRADES CLOSE TOMMORW Which side lengths form a right triangle?

Answers

Letter A is right


Explanation: best of luck

Answer: B

Step-by-step explanation:

3√3 / √30
step by step pls

Answers

Answer:

3√10/10

Step-by-step explanation:

see the photo

............

Wut that guy said it’s right

helpppp plzzz...plz give steps

Answers

Answer:

Volume of hemisphere =

[tex] \frac{2}{3} \pi \: r {}^{3} [/tex]

Given volume =

[tex]18\pi \: cube[/tex]

[tex] \frac{2}{3} \times \frac{22}{7} \: {r}^{3} = 18 \times \frac{22}{7} [/tex]

So pi is there on both the sides so it will be cut from both LHS and RHS

[tex] \frac{2}{3} \times r {}^{3} = 18[/tex]

[tex]2 \times r {}^{3 } = 18 \times 3[/tex]

[tex]2 \times r {}^{3} = 54[/tex]

[tex]r {}^{3} = \frac{54}{2} [/tex]

[tex]r {}^{3} = 27[/tex]

[tex]r = 3[/tex]

Therefore value of r=3 inches

If you are helped with my answer pls tag me brianliest I really want it... Plzz

THANK YOU

R=3 inches that’s the answer

Hey PLZZZZ HELP!

The profits earned in a year by some companies in a city are shown below: Company Profit A $160,000 B $140,000 C $130,000 D $1,120,000 E $150,000 Based on the data, should the mean or the median be used to make an inference about the profits earned by these companies? Mean, because it is in the center of the data Median, because it is in the center of the data Mean, because there are no outliers that affect the mean Median, because there is an outlier that affects the mean

Answers

Answer:

mean because there is an outlier

Step-by-step explanation:

since there is an outlier that affected the mean which is company D, it can only be mean because the outlier is too big.

Mean. It gives a better idea of the profits.

pls help im failing lol Which side lengths form a right triangle?

Answers

Answer:

4 4 8

Step-by-step explanation:

b

Answer:

A

Step-by-step explanation:

i did the math and it took forever please mark brainliest :)

find surface area of the square pyramid!!! pls i need help with explaining!!!!!

Answers

Answer:

39 square yards

Step-by-step explanation:

The base is a square, so the area is 3 x 3 = 9.

Each side is a triangle, and the formula for that is (1/2)(base)(height)

The base of each side is 3 and the height of each is 5. There are 4 of those, so it's 4 x (1/2)(3)(5) = (2)(3)(5) = 30

Add the base and sides

30 + 9 = 39

Answer:

39 yards

Step-by-step explanation:

surface area = A + [tex]\frac{1}{2}[/tex] ps

A = area of the base

p = perimeter of the base

s = slant height

the base of the pyramid is a square, all sides and angles are equal

; 3 yards

; area of a square  = l x w

; area of the base = 9 yd (A)

perimeter of the base ; 3 + 3 + 3 + 3 = 12 yd (p)

slant height = 5 yd (s)

; sub it into the formula

surface area  = 9 + [tex]\frac{1}{2}[/tex] x 12 x 5

surface area = 39 yd

Help with math please earn 10 points

Answers

I think it’s the second one

Answer:

Option 3: y = -0.5 + 27

Step-by-step explanation:

Where the + 27 comes from: It comes from where the line starts

Negative slope always goes down, and has negatives. Hence the answer is 3.

For each pair of points, think about the line that joins them.

For which pair(s) is the line parallel to the x-axis?

Select all correct answers.

Answers

Answer:

fd

Step-by-step explanation:

spañol

Answer:

The answer might be number b

How do I solve this?

Answers

Answer:

We know the width maybe 22ft and at its widest.

IF we say 3 triangles can be fit into a polygon given 2 sides then we 180 x 3 = 560 degree if its 3 rectangles then if it is easier to make 4 rectangles if square shape we can subtract the smaller measures of the missing rectangle.

With triangles;

The number of triangles is always 2 less than that that of its sides.

= 3 triangles + 2 = 5 sides.

The sum of interior angles

(n-2) x 180 = 540 degree

5-2 x 180

To determine the amount of sides; we remember

(n-2) 180

360 = (n-2) 180

+360 = 180 -360 +180

+360 = 180 = (3 triangles with 180)

900/180 = 5 sides.

If we say 5 triangles created with 3 given lengths

then same rule applies n-2 (180)

=7 sides

= if regular shape largest line 22 x 7 = 154 ft sq = Area.

Answer:

374

Step-by-step explanation:

First do the area of the triangle 7 x 22 = 308 308/2 = 154 22 x 10 = 220 220+154 =374

Plz help will be marked BRAINLIEST!
Thanks

Answers

Answer:

a) a + c = 90

d) a + b + c + 30 = 180

e) The angles marked a and b are vertical

f) The angles marked a and c are complementary.

Step-by-step explanation:

A complementary angle is and angle that has a sum of 90 degrees. A vertical angle is 2 angles facing each other, that are completely identical. 180 degrees is the sum of a strait line. 90 degrees is a right angle.

Hope this helps! Have a wonderful rest of your day!

-kiniwih426

kini is correct, have a good day/night !!





-bianca

solve the following equaion.

6x - 4 = 2 + 3x

Answers

6x-4=2+3x
6x-4-3x=2
6x-3x=2+4
3x=2+4
3x=6
so your answer would be x=2

6x-4=2+3x

=> 6x-3x=2+4

=>3x = 6

=> x = 6:3

=> x = 2

Which two statements about the volumes of the prisms are true?

Answers

The answer for this question is A
Answer: A
Explanation: when you solve it

please help me with the questions below

Answers

Answer:

2) 3/4 of the girls did not have 1 piece bathing suits

3) 4/6 were at the kiddie pool

4) 1/3 was boys

sorry if this was wrong

3/4 of the girls did not have a bathing suit. Since 1/4 of the girls did that would leave 3 out of the four left.

I NEED THIS ASAP!!!!!!!!

Answers

Answer:

A

Step-by-step explanation:

A: -a/-b=c (negative number divided by negative number is positive number A is correct)

B: -a x b =c (negative number times positive number is become a negative number so correct answer is -c B is wrong)

C: -a/b=c (negative number divided by positive number is become a negative number so correct answer is -c C is wrong)

D: -a x -b=-c (negative number times negative number is become a positive number so correct answer is c D is also wrong)

Please help!!!!!!!!!!!!!!!!!!!!

Answers

1. Corresponding

2. Angle 8
the first one is corresponding
the second one is angle 8
Other Questions
What is result driven Resuelvas las ecuaciones de primer grado Plz help will be marked BRAINLIEST Thanks Please help !!!!!!!!!!!!!!!!! The organ that controls your internal clock is called... Can anyone help me with a mental ability work plzz Child abuse, domestic violence, and juvenile delinquency describes the _________ ________ in our society. * 1 point Social group Cultural History Cultural Identity Social Issues Payment of an above-market wage reduces shirking by employees and reduces worker turnover because it multiple choice 2 decreases worker productivity. raises the opportunity cost of losing a job. lowers the opportunity cost of losing a job. creates more supervisory positions. pagkakaroon ng oras para magdasal at magpunta sa pook dalanginan? Ill mark brainliest Choose the from of the verb that best complete the sentence What angle is complementary to 2 ? Traci spent hour eating breakfast and hour helping clean up after breakfast.How much time did she spend in all?Give your answer in simplest form.hourEnterPlz help I I dont understand this could someone try its geometry Use the diagram to find cos x as a fractionin simplest form. helpppp plzzzzzzzzz.....also give steps 20 divided by 34 HEEEEELPPPPPPPPPPP introduction: connect each word with the phrases that best defines it. Cary sets up a checking account with an initial balance of $27,700, and the rent for herapartment is deducted every month for a year. After a year the balance is $7900(Assume no other transactions occur on the account) Find the missing angle